PT14.S4.Q9 - anyone who supports the new tax plan

lsatjourneylsatjourney Member
edited February 2018 in Logical Reasoning 207 karma
I'm really having trouble with this question. Am I supposed to approach it as a sufficient assumption question though it's a flaw question? Thanks in advance!

Comments

  • BinghamtonDaveBinghamtonDave Alum Member 🍌🍌
    edited November 2016 8694 karma
    This is a remarkably difficult question but one in which given a bit of light, makes sense. Timed, at this stage in my prep, I get this question wrong 99/100. Untimed, the logic is some of the most complex I have come across on an LR stimulus. The question also requires of us an understanding of our common argument forms.

    This is what we know:
    -1. "If you support the new plan then you have no chance of getting elected."
    -2. "If you understand econ then you wouldn't support the new plan."
    Those are two really quite simple conditional statements: lesson thus far is to remember that "anyone who" is sufficient indicator.

    Conclusion:
    -3."If Chance of elected then understand econ."
    Lesson this far is to parse out that this is the conclusion, it follows the comma after the sentence that starts with "since" meaning that support is given for it. Pretty subtle.

    Now lets map this out:
    1.Support--->No chance/Chance--->~Support
    2.Understand--->~Support/Support--->~Understand

    3.Conclusion:
    Chance--->Understand

    How did they arrive at this conclusion? Well nothing above allows us to arrive at this conclusion through valid reasoning. Any way we try it, we cannot get that conclusion. What now? Well, take a look at the conclusion: Chance--->Understand. We have those terms in our premise, what the author of this must be trying to say is that Chance--->~Support---->Understand: A--->B---->C, but in this variation is a sufficient necessary flaw.

    I realize how tough that sounds, it took me 27 minutes to get. Consider an example:
    If something is a cat, it isn't a fish
    If something is a donkey it isn't a fish

    Therefore if something is a cat, it is a donkey.

    What have I done there? I have reversed the sufficient and necessary relationship of the second premise. As the author of that phrase, I have done the internal logic as:
    Cat---->~Fish
    ~fish--->Donkey

    Combined the sentences and arrived at
    Cat--->Donkey




    Now here comes the most difficult part, and again, something we can consult the lessons on. The answer choices are all (bizarrely) references to "some" statements. Why? Well, some statements can be used to deny a conditional relationship. What are we trying to deny? We are trying to deny the ~support--->understand. How do we do this? We say that there are some people that ~Support and ~understand. This is what (D) says.

    I don't say this lightly, I have put in 12 months into studying for this exam, this is one of the only questions that I have come across that I consider unfair. I think it demands too much from perspective law students. I have never invoked that before.
  • AlexAlex Alum Member
    23929 karma
    @David3389 said:
    This is a remarkably difficult question but one in which given a bit of light, makes sense. Timed, at this stage in my prep, I get this question wrong 99/100. Untimed, the logic is some of the most complex I have come across on an LR stimulus. The question also requires of us an understanding of our common argument forms.
    I checked this question out and it was truly difficult.

    Awesome and informative explanation though, @David3389 !
  • lsatjourneylsatjourney Member
    207 karma
    Thank you! Great explanation!
  • doyouevenLSATdoyouevenLSAT Core Member
    609 karma

    how often does one like this one show up? later....

  • Leah M BLeah M B Alum Member
    8392 karma

    Sounds like you already have your question answered, but I looked at this and started formulating my own way to explain it, might as well type it out. (Also, I started literally thinking through an explanation in JY's voice hahaha. He's officially permeated my brain.)

    This definitely is a tough one. And to answer @doyouevenLSAT, I don't think questions are really framed this way much (if at all) any more. This is from a very old test.

    The way this broke down for me is envisioning two groups of people: those who support the tax plan, and those who don't.

    We know that the people who support the tax plan are completely unelectable. It's a terrible plan, I guess. So they are ruled out from ever being elected.

    On the other side, we have the people who do not support the tax plan; they are the only electable ones. We also know that if someone truly understands economics, then they are in this group of people who don't support the plan. However, does that mean that every person who doesn't support the tax plan truly understands economics? No, that's not what we were told. Basically, we were told this:

    Understand econ --> Support tax plan

    And the flawed conclusion is stating this:

    Support tax plan --> understand economics

    The flawed conclusion we are given is that if you are electable (meaning: must not support the tax plan) then you also understand economics. But what if you don't know anything about economics, you just have a gut feeling that it's a crappy tax plan? According to the premises we were given, you could still be elected. All we know is that people who support the tax plan can't be elected. Anyone who doesn't support it is still eligible, regardless of your knowledge of economics.

    Therefore, the conclusion is flawed because it ignores the people who D) don't understand economics but also don't support the tax plan.

  • doyouevenLSATdoyouevenLSAT Core Member
    edited January 2018 609 karma

    @"Leah M B" thanks a lot. I honestly had damn near panic attack reading this. it was like none of what i learned was helping me with this. but breaking it down into like abc's made it easier to see what was happening.

    i ended up labeling it "most difficult LR question ever"

  • BinghamtonDaveBinghamtonDave Alum Member 🍌🍌
    8694 karma

    In my experience the form is not that common, but the way in which the answer choice is formulated is more common. Earlier this week I did PT 46 and there is a flaw question on the third section (23) in which the test writers took a common flaw, hid the common flaw and then obscured the way in which they described the flaw.

  • Leah M BLeah M B Alum Member
    8392 karma

    @BinghamtonDave said:
    In my experience the form is not that common, but the way in which the answer choice is formulated is more common. Earlier this week I did PT 46 and there is a flaw question on the third section (23) in which the test writers took a common flaw, hid the common flaw and then obscured the way in which they described the flaw.

    Interesting, yeah essentially this breaks down to being a confusion of necessary and sufficient, which is extremely common. But the way it's written in this one strikes me as pretty unusual.

    Then again @doyouevenLSAT, the LSAT folks do like to throw wrenches in with extremely difficult questions. I took the recent December test and there were at least 3 questions on it that I would label very, very difficult. Not flaw questions though. 2 of the 3 I sort of (mostly?) figured out on test day, and make sense to me now. The 3rd question took me a lonnnng time of reading and re-reading it to even figure out what it was asking or how the answer choices even related to it. Ugh.

    So I guess my answer is more like, I don't recall seeing this particular way of disguising the answer, but those dang LSAT people are a bunch of tricksters. Haha.

  • BinghamtonDaveBinghamtonDave Alum Member 🍌🍌
    8694 karma

    I've been really looking into sufficient/necessary flaws lately. A particularly difficult way in which the test writers described the flaw appears on PT A section 4 question 20.

Sign In or Register to comment.